Jiasheng Jin
Jiasheng Jin
  • 214
  • 169 831
Number theory problem | 133^5+110^5+84^5+27^5=n^5
You can leave a comment down below,
like and subscribe to my channel if you like my content.
You can also support me on Ko-fi at ko-fi.com/jiashengjin
Переглядів: 280

Відео

Rough estimation of 1+2^2+3^3+…+n^n
Переглядів 16928 днів тому
You can leave a comment down below, like and subscribe to my channel if you like my content. You can also support me on Ko-fi at ko-fi.com/jiashengjin
When is this an integer? n^1/n-7 | Swedish Math Olympiad 2002
Переглядів 2,4 тис.Місяць тому
You can leave a comment down below, like and subscribe to my channel if you like my content. You can also support me on Ko-fi at ko-fi.com/jiashengjin
Some hard analysis | limit of (an+1+an+2)/an
Переглядів 407Місяць тому
You can leave a comment down below, like and subscribe to my channel if you like my content. You can also support me on Ko-fi at ko-fi.com/jiashengjin
Iran Math Olympiad number theory problem | x^13 equals 21982145917308330487013369
Переглядів 27 тис.Місяць тому
You can leave a comment down below, like and subscribe to my channel if you like my content. You can also support me on Ko-fi at ko-fi.com/jiashengjin
Does this limit exist? | sqrt(1+sqrt(2+…+sqrt(n)))
Переглядів 339Місяць тому
You can leave a comment down below, like and subscribe to my channel if you like my content. You can also support me on Ko-fi at ko-fi.com/jiashengjin
A floor function equation | floor of sqrtn+1/2 floor of sqrt(n-3/4)+1/2
Переглядів 132Місяць тому
You can leave a comment down below, like and subscribe to my channel if you like my content. You can also support me on Ko-fi at ko-fi.com/jiashengjin
IMO 2020 Question 2
Переглядів 287Місяць тому
You can leave a comment down below, like and subscribe to my channel if you like my content. You can also support me on Ko-fi at ko-fi.com/jiashengjin
Harmonic series | generalized Abel second theorem
Переглядів 168Місяць тому
You can leave a comment down below, like and subscribe to my channel if you like my content. You can also support me on Ko-fi at ko-fi.com/jiashengjin
Tricky inequality | double sum (aiaj/ci+cj) (bibj/ci+cj) bigger than (aibj/ci+cj)^2
Переглядів 145Місяць тому
You can leave a comment down below, like and subscribe to my channel if you like my content. You can also support me on Ko-fi at ko-fi.com/jiashengjin
Some integral inequality
Переглядів 213Місяць тому
You can leave a comment down below, like and subscribe to my channel if you like my content. You can also support me on Ko-fi at ko-fi.com/jiashengjin
A mysterious floor equation identity
Переглядів 162Місяць тому
You can leave a comment down below, like and subscribe to my channel if you like my content. You can also support me on Ko-fi at ko-fi.com/jiashengjin
Last 2 digits of 9^9^9^9
Переглядів 282Місяць тому
You can leave a comment down below, like and subscribe to my channel if you like my content. You can also support me on Ko-fi at ko-fi.com/jiashengjin
How on earth can we show the convergence radius is 1?
Переглядів 131Місяць тому
You can leave a comment down below, like and subscribe to my channel if you like my content. You can also support me on Ko-fi at ko-fi.com/jiashengjin
MIT Integration Bee 2024 problem 4
Переглядів 198Місяць тому
You can leave a comment down below, like and subscribe to my channel if you like my content. You can also support me on Ko-fi at ko-fi.com/jiashengjin
3 ways to solve this MIT Integration Bee problem | cosx+xsinx/x(x+cosx)dx
Переглядів 456Місяць тому
3 ways to solve this MIT Integration Bee problem | cosx xsinx/x(x cosx)dx
A fabulous integral from MIT Integration Bee | infinite nested roots
Переглядів 1,4 тис.Місяць тому
A fabulous integral from MIT Integration Bee | infinite nested roots
A problem from Analysis | limsupn((1+an+1/an)-1) no less than 1
Переглядів 2272 місяці тому
A problem from Analysis | limsupn((1 an 1/an)-1) no less than 1
How this hard infinite sum equals pi/2ln2
Переглядів 4272 місяці тому
How this hard infinite sum equals pi/2ln2
A tricky integral from 0 to pi/2 of lnsinxdx
Переглядів 1572 місяці тому
A tricky integral from 0 to pi/2 of lnsinxdx
Is it 7 or any number is correct?
Переглядів 2532 місяці тому
Is it 7 or any number is correct?
Integral of dx/1+acosx
Переглядів 2072 місяці тому
Integral of dx/1 acosx
This cannot be true! | What went wrong?
Переглядів 1172 місяці тому
This cannot be true! | What went wrong?
A tricky limit problem | lim(a2n+2an)=0
Переглядів 2572 місяці тому
A tricky limit problem | lim(a2n 2an)=0
No integral by parts allowed | integral from 0 to 1 of lnxdx
Переглядів 7102 місяці тому
No integral by parts allowed | integral from 0 to 1 of lnxdx
Some inequalities of liminf and limsup
Переглядів 1552 місяці тому
Some inequalities of liminf and limsup
A monster integral | (tanx)^1/3dx
Переглядів 5192 місяці тому
A monster integral | (tanx)^1/3dx
A mad integral | x^2/(xsinx+cosx)^2dx
Переглядів 3162 місяці тому
A mad integral | x^2/(xsinx cosx)^2dx
Pi equals 4? | path lengths equal each other?
Переглядів 1142 місяці тому
Pi equals 4? | path lengths equal each other?
Putnam 2018 A1 | 1/a+1/b=3/2018
Переглядів 1,1 тис.2 місяці тому
Putnam 2018 A1 | 1/a 1/b=3/2018

КОМЕНТАРІ

  • @Alex-ri6be
    @Alex-ri6be 9 днів тому

    I like the use of that lemma.

  • @prism223
    @prism223 11 днів тому

    I actually guessed the correct answer using a few memorized logarithms: The base ten logs for 1-10 are approximately 0, 0.3, 0.48, 0.6, 0.7, 0.78, 0.85, 0.9, 0.95, 1 Looking at the big number, it's basically 22x10^24, so its base 10 log is 24 + log(11) + 0.3. The 11 log isn't in the list I memorized, so I used linear interpolation between 12 and 10: log(12) ~ 1.08, so log(11) ~ 1.04. Then log(big number) ~ 25.34. Dividing by 13 to get the 13th root logarithm, I just do 3 digits mentally to get 1.94. This is extremely close to what I would compute for log(90) ~ 1.95, so my first guess would be that the 13th root of big number is 89.

  • @amirm1266
    @amirm1266 13 днів тому

    Hello from iran

  • @RealQuInnMallory
    @RealQuInnMallory 14 днів тому

    (x ➖ 3x+2)

  • @lechaiku
    @lechaiku 15 днів тому

    Another approach without using modulo and log (this is "no-pen-no-paper" method): 1. the last digit of x must be 9, because only 9 goes in cycles (9,1) with the 13th power being 9. 2. 100^13 is 27-digit number, so x must be 2-digit number ----> 26(digits) : 13 = 2. 3. x can't be 99, because the given 26-digit number is too small (it's obvious). 4. 8 = (2^3)^13 = 2^39; let's calculate 2^40 5. 2^10 = 1024, let's use "1000" 6. 2^40 = approx. 8^13 = approx 1000 ^4 7. 80^13 = approx 1000 ^4 * 10^13 = 10^12 * 10^13 = 10^25 8. It is less than the given 26-digit number 9. therefore x = 89

  • @AnmolSahu
    @AnmolSahu 19 днів тому

    1. Since 100^23 has 27 digits, x must be a two digit number. 2. Last digit of x^(4*3+1) must be same as last digit of x^1, therefore, last digit of x must be 9. 3. Digital sum of the given number is 8, therefore, the digital sum of x^13 must be 8. Solving which we get x=89.

  • @krule3753
    @krule3753 22 дні тому

    i think you can stop and show contradiction at M(1/2)^4 < M(1/2)^k, since from the observation its implied that k > 4 but obviously in that case the inequality does not hold.

    • @asdasdasasdas5837
      @asdasdasasdas5837 21 день тому

      not really we have a_n<M for every n for example choose epsilon=1/2 then a_n<1/2(a_n+1 + a_n+2) but this doesnt imply M<1/2*(M+M) or M<M I hope you understand where im going with this, i really want to articulate more but english is my second language and i dont really know how to put it

  • @sie_khoentjoeng4886
    @sie_khoentjoeng4886 22 дні тому

    Value of X must be odd, since the result is odd, and X is not 1 or 5 (where the result has unit 1 and 5) For K=1,2,3,4,5,..., then ?3^K, the unit result is 3,9,7,1,3,9,7,1,... ?7^K, the unit result is 7,9,3,,1,7,9,3,1,... ?9^K the unit result is 9,1,9,1,9,1,9,.... For K=13, then ?3^K has unit 3, ?7^K has unit 7 and ?9^K has unit 9 So, possible answer is 09,19,29,39,49,59,69,79,89,99 Number of digit 99^13 is about 36 (less than 36) so 99 is rejected, then X=88 (** if there is no typo like this 21,982,145,917,038,330,487,013,369 😊😊😊)

  • @tylosenpai6920
    @tylosenpai6920 23 дні тому

    I just thought... Ends with 9 -> The original number must end with 3, 7, or 9 3 -> Final digits of 3 9 7 1 that repeats itself, will be 3 in 13 (Fail) 7 -> Final digits of 7 9 3 1 that repeats itself, will be 7 in 13 (Fail) 9 -> Final digits of 9 1 that repeats itself, will be 9 in 13 (Success) There are 26 digits, close to 10^26 with 27 digits (AKA 100^13), but not too close I'm now left with 89 and 99 as the most likely answers, and thought, "It shouldn't be too close" and picked 89...i was right...

  • @user-kr6us9gx6z
    @user-kr6us9gx6z 24 дні тому

    Last digit is 9, it's easy. 80^13 < 10^25, so it can be 99 or 89. If it 99 then the given number mast be divided by 11. Easy to se that it doesn't. So it is only 89

  • @orionspur
    @orionspur 27 днів тому

    Rather easy to check that by sheer size x it has to be between 80 and 90 and has to end with 3 7 or 9. Of which only 9 can work. So x is 89.

  • @samueldeandrade8535
    @samueldeandrade8535 27 днів тому

    For people who doesn't know, this equation was the first counterexample, to Euler's conjecture: If the sum of n many kth powers of positive integers is itself a kth power, then n≥k This conjecture was proved wrong in 1966, by L. J. Lander and T. L. Parkin. Their example was 27⁵+84⁵+110⁵+133⁵ = 144⁵

  • @ravTubeable
    @ravTubeable 27 днів тому

    0:08 X is a real number 0:18 X is an integer number WTF!? How did you figured out (in just 10 seconds) that solution is an integer number?

  • @samueldeandrade8535
    @samueldeandrade8535 27 днів тому

    Despite your content not being serious, it is very enjoyable to watch you.

  • @samueldeandrade8535
    @samueldeandrade8535 27 днів тому

    Hahahahahahaha.

  • @meadbert
    @meadbert 27 днів тому

    I counted digits and figured it had to be 2 digit number. Last digit had to be 9. Then I compared to 100 and saw we ended up at 22% of what 100 would have. This means we haved a little over two times so a half life of roughly 6 years. Law of 72 says we are decreasing by 12% per year to have a half life of six years so answer should be close to 88. Since last digit is 9 the answer must be 89.

    • @notmymain2256
      @notmymain2256 25 днів тому

      Oh no finance has tainted modular arithmetic

  • @havenfractal
    @havenfractal 28 днів тому

    10<x<100, since 10^13 would be too small and 100^13 is too big. x cannot end with an even, 1, or 5, because then x^13 would end in an even, 1, or 5, respectively. 7^2 and 3^2 end in 9, so 7^4 and 3^4 end in 1, so 7^12 and 3^12 end in 1. So 7^13 ends in 7 and 3^13 ends in 3. So the last digit cannot be 7 or 3. 9^2 ends in 1, so 9^12 ends in 1. So 9^13 ends in 9. The last digit can be 9. So we know that x is in the form 10y+9, with 0<y<10. Looking at the first 13 digits, y^13 <= 2198214591730, or about 2.198 * 10^12 because if it was bigger, the most significant 13 digits would have to be bigger. 7^2 < 50, so 7^12 < 50^6. So 7^13 < 50^6*7, or 1.09375 * 10^11. This is far too small. 8^3 = 2^9 > 500. So 8^12 > 500^4, and 8^13 > 5^4 * 8 * 10^8. So 8^13 > 5*10^11, which is in the right ballpark. 9^2 = 81 > 80, and 9^13 > 80^6 * 9 = 8^6 * 9 * 10^6 > 500^2 * 9 * 10^6, or 25*9 * 10^10, or 2.25*10^12, which is too big. So y=7 is proven too small, y=9 is proven too big, and y=8 looks plausible. So the answer is 89.

  • @user-bk1wl9ez1c
    @user-bk1wl9ez1c 28 днів тому

    Прикольно

  • @cloudy7989
    @cloudy7989 29 днів тому

    Strangely, x^13 = my phone number multipled by my wife's number plus our door number to the power of the numbers in my post code.

  • @lucabattazzi
    @lucabattazzi 29 днів тому

    Thank you very much for this proof. There aren't many out there actually proving the existence of the limit, by using lim inf and lim sup

  • @shashiplaysroblox8622
    @shashiplaysroblox8622 29 днів тому

    Gonna guess like 91?

  • @amarnathprasad9986
    @amarnathprasad9986 29 днів тому

    Sir, doesn't this depend on the choice of e? We could choose e>1/2, and then a_n <= M(2e)^k -> inf, implying a_n is unbounded. Am I missing something here?

    • @krule3753
      @krule3753 22 дні тому

      it should hold for ALL epsilon > 0, so no matter what epsilon we pick it should hold. since it doesn't work for epsilon = 1/4, then the "all" part doesn't hold, contradiction.

    • @amarnathprasad9986
      @amarnathprasad9986 18 днів тому

      @@krule3753 Ohh, I see... Thanks for the explanation.

  • @Jiuge2004
    @Jiuge2004 Місяць тому

    It's obviously 89. Because 9^9 has the last digit of 89 and same as 9^(10n+9). Which means that 9^9^9^9=9^9^(....89)=9^(....89)=.....89.

  • @Jiuge2004
    @Jiuge2004 Місяць тому

    89

  • @atharvanaik5966
    @atharvanaik5966 Місяць тому

    some classic hard anal. great video

  • @umarus2
    @umarus2 Місяць тому

    Can we split fraction to _a_(n+1) / a_n_ and _a_(n+2) / a_n_ so at least one of them goes to infinity? Then it may be easier to prove that _a_n_ is unbounded if _a_(n+1) / a_n_ is unbounded.

  • @jyb.el2010
    @jyb.el2010 Місяць тому

    It doesn't go neither from the paper...

  • @pesilaratnayake162
    @pesilaratnayake162 Місяць тому

    I used 10^13 < x^13 < 10^26 -> 10 < x < 100. Then n^k (mod 10) == n^(k (mod φ(10)) -> n^13 == 9 (mod 10) == n^(13 (mod 4)) -> n == 9 (mod 10) From there, a binomial expansion gives 90^13 = 10^13 * (10-1)^13 ≈ 10^26 - 13*10^25 + 78*10^24 = 52*10^25, whereas 10^26 - 13*10^25 + 78*10^24 - 286*10^23 = 23.4*10^25. The remaining terms are alternating and are decreasing in magnitude, so 90^13 > 2.34×10^26. Testing 80^13 tells us that 80^13 < 10^26 - 26×10^25 + 312×10^24 - 2288×10^23 + 11440×10^22 ≈ 0.3×10^25, which is too small (calculations might be a little off since I did them in my head). Therefore, since 80<x<90, and x==9 (mod 10), then if x € Z, then x = 89. I like that your way gives a small set of values that we can test, though.

  • @Loots1
    @Loots1 Місяць тому

    x = 13th root of the original number *drops chalk as explosion destroys board*

  • @dimitrisanasto1168
    @dimitrisanasto1168 Місяць тому

    Great video sir! What is the source of this exercise? Thank you.

  • @samueldeandrade8535
    @samueldeandrade8535 Місяць тому

    I am starting to suspect Mr. Jiasheng Jin is playing with us. Because this is too much for this question ... The first natural thing to do is to try some numbers. And we "immediately" can see that for n=8, we have 8^(1/1) = 8 for n=9, we have 9^(1/2) = 3 For n=10, 10^(1/3) is NOT an integer. For n≥11, we have n-7≥4, then n < 2^(n-7) which implies n^(1/(n-7)) < (2^(n-7))^(1/(n-7)) < 2 Clearly, n^(1/(n-7))≠1, so the obtained inequality implies 8 and 9 are the only solutions. No need to use GM ≤ AM, or to manipulate fractions, etc ...

    • @justpotatoit
      @justpotatoit 28 днів тому

      That is also what I thought

    • @maxcanaday6208
      @maxcanaday6208 27 днів тому

      Can you please explain what importance is n < 2^(n-7)? I don't know how you get that from "For n≥11, we have n-7≥4."

    • @samueldeandrade8535
      @samueldeandrade8535 27 днів тому

      @@maxcanaday6208 the importance of n < 2^(n-7) is that it can be used to conclude n^(1/(n-7)) < 2 The part "n≥11, then n-7≥4 and n<2^(n-7)" is just my way to invoke an induction, because for for n=11, 11<16=2^4 ✓ for n≥11, if n<2^(n-7), then n+1 < 2n < 2*2^(n-7) < 2^((n+1)-7) In order to not write all that, I just indicated the first case the inequality works. I could have just "n-7≥4", but I guess I wanted to make it clear the exponent 4 that appears in the first case.

  • @atnm
    @atnm Місяць тому

    wow this is crazy

  • @LeDuyBao-ex7fj
    @LeDuyBao-ex7fj Місяць тому

    Thanks for the interesting problem!

  • @howareyou4400
    @howareyou4400 Місяць тому

    This is way too complicated. It's a super easy problem: 1. check the last digit, it can only be 9 2. check the remainder mod 9, it's 8 3. check the total digit count, it can only have two digits therefore it's 89

    • @notmymain2256
      @notmymain2256 25 днів тому

      Ye lol, last digit 9 is also trivial by mod2 and mod5; I guess crt rocks again

  • @Dalroc
    @Dalroc Місяць тому

    A) Last digit of x^(4n+1) is the same as the last digit of x. So we know that the last digit of x has to be 9. B) 100^13 = 10^26, which is 27 digits long, so we know that x is less than 100. C) Fermat's little theorem tells us that a^p = a (mod p) for any prime p. Dividing the given number by 13 gives a remainder of 11. From A, B and C we know that we're looking for a number of the form 13k + 11 < 100 that ends on a 9. This means that 13k has to end on an 8. Looking at the times table for 13 find 78 = 13*6 as the only one that ends on an 8. 78 + 11 = 89. x = 89

    • @Felipe-sw8wp
      @Felipe-sw8wp Місяць тому

      Very succinct and nice answer. Excellent.

    • @LarifariRambazamba
      @LarifariRambazamba Місяць тому

      I used A and B like you did, but not C. Since x¹³ ≈ 2.2 × 10²⁵ and 100¹³ = 10²⁶, we know that x must be a bit below 100, but 99 seems too close, so we hypothesize that it is 89. Since 89 is prime (9*10=90, 7*13=91), then if we can show that 89 divides x¹³ then 89 must divide x, which would mean that x = 89 because any multiple would be greater than 100. There is a relatively easy way to check for divisibility by (10n−1) which avoids long division and only uses multiplication and addition. n*(10a + b) ≡ (a + nb) mod (10n−1) and n ≢ 0 mod (10n−1) so (10a + b) ≡ 0 mod (10n−1) iff (a + nb) ≡ 0 mod (10n−1) For 89 we have n=9. So if we start with any number A, then let A′ be the new number formed by erasing the least significant digit of A, and let A″ be 9 times the least significant digit of A added to A′. Then A ≡ 0 mod 89 iff A″ ≡ 0 mod 89. Apply this recursively. We get the sequence: 21982145917308330487013369 (this is x¹³) 2198214591730833048701417 219821459173083304870204 21982145917308330487056 ... 21982199 2198300 2225 267 89 And because the last number is divisible by 89, then all the previous numbers in the sequence are also divisible by 89, including the original number which is x¹³. But that means that x=89, as mentioned in the first paragraph.

  • @HartmutRick
    @HartmutRick Місяць тому

    Once you figured out that 1001 is a multiple of 13, you can calculate x^13 mod 1001 by subtracting multiples of 1001 until the remaining number is smaller than 1001. For example, subtracting 1001*k*10^n can be achieved by subtracting k simultaneously from the 10^n and the 10^(n+3) digits, i.e. you can subtract (or add) the same amount simultaneously from any two digits that are 3 positions apart from each other. This does not change the remainder when dividing by 1001 (or 13). For example, if x^13 starts with the digits 21982145..., subtract 2 from the first and the 4th digits, you obtain 1962145.... Then subtract 1 from the 1st and the 4th digit to obtain 961145... Now next is to subtract 9 from the 1st and 4th digit. If the 4th digit is not large enough, include the 3rd digit: 60245... If you keep doing this 21 times you end up with the remainder 89. In principle this is just a long division of x^13 by 1001, but the calculation is not difficult, and you don't need to keep track of the result of the division.

  • @HartmutRick
    @HartmutRick Місяць тому

    If we know that x is an integer: x^13 has 26 digits, so x must have 2 digits. x^13 is an odd number, thus x also has to be odd. The last digit of x cannot be 1 or 5, because then all powers also end in 1 or 5, respectively. Thus the last digit of x must be 3,7 or 9. 3^2 and 7^2 both end in 9, thus 3^4 and 7^4 end in 1, as well as 3^12 and 7^12. Thus 3^13 and 7^13 end in 3 and 7, respectively. Thus the last digit of x must be 9. It is easy to calculate x^13 mod 3, just add all digits and we find that x^13 mod 3 = 2. This means x mod 3 = 2 as well. We can exclude 0 and 1, because if x mod 3 was 0 or 1, then all powers of x mod 3 are also 0 or 1, respectively. Now we know the last digit of x is 9, and x mod 3 = 2. A possible value is x=89. The next smaller value is 59. x must be a 2-digit number, but it cannot be too small. 60^13 has already fewer than 26 digits: 60^2=3600, 60^4=36^2*10^4=1296*10^4. This is smaller than 13*10^6, thus 60^8 is smaller than 169*10^12, which has only 15 digits, so we already lost one digit, 60^13 can have at most 25 digits, most likely it has not more than 24. This leaves us with the only possible solution of x=89, since x=59 is already too small.

  • @r75shell
    @r75shell Місяць тому

    in the beginning of the video you say x from real numbers. then you can't assume it's an integer

    • @phlopmeistergenerel
      @phlopmeistergenerel Місяць тому

      Real numbers encapsulate the integers in this case

    • @r75shell
      @r75shell Місяць тому

      @@phlopmeistergenerel find x^3 = 10000, x from real numbers. Then apply all logic as in the video (assuming it's integer): x^3 = x (mod 3). then 10000 mod 3 = 1, then x mod 3 = 1. Possible solutions 1,4,7,10,13,16,19,22,25,28. Because 30^3 = 27000 > 10000 1 obviously doesn't work 4^3 = 4 (mod 10) bad 7^3 = 3 (mod 10) bad 10^3 = 0 (mod 10) looks good 13^3 = 7 (mod 10) bad 16^3 = 6 (mod 10) bad 19^3 = 9 (mod 10) bad 22^3= 8 (mod 10) bad 25^3 = 5 (mod 10) bad 28^3 = 2 (mod 10) bad the only option which gives correct last digit is 10. *THUS* answer is 10. This is wrong because we assumed x is integer. Similarly, it's not the fact that your answer is correct. Because *you have to raise answer into 13 power without calculator* to check is it an answer or not. This invalidates requirement to find an answer without using calculator.

  • @balthazarbeutelwolf9097
    @balthazarbeutelwolf9097 Місяць тому

    If you give the 'a' series 2 indexes, and define a_k0 = sqrt(k) and a_{k,n+1}=sqrt(k+a_{k+1},n. Then you can prove by induction on n that a_kn<n+1, and the whole shabang follows.

  • @samueldeandrade8535
    @samueldeandrade8535 Місяць тому

    Use the full cycle of 10 (mod 13), making no additional divisions by 13. We have 10¹ = -3 (mod 13) 10² = -4 (mod 13) 10³ = -1 (mod 13) 10⁴ = 3 (mod 13) 10⁵ = 4 (mod 13) 10⁶ = 1 (mod 13) The number is 21 982145 917308 330487 013369 Just sum the digits in each column and multiply by the respective value of 10ⁿ, n=0,...,5, (mod 13), (9+9+3+0)×4 + (8+1+3+1)×3 + (2+7+0+3)×(-1) + (1+3+4+3)×(-4) + (2+4+0+8+6)×(-3) + (1+5+8+7+9)×1 = 21×4 + 13×3 + 12×(-1) + 11×(-4) + 20×(-3) + 30×1 Mod 13: = (-5)×4 + 0×3 + (-1)×(-1) + (-2)×(-4) + (-6)×(-3) + 4×1 = (-5+2)×4 + 0 + 1 + + 18 + 4 = -12 + 1 + 5 + 4 = 1 + 10 = 11 No division of three digit numbers or 6 digit numbers required. It has to end in 9. So 13k+11 = ...9 This means k ends in 6, so k = 10n+6, 13k+11 = 130n+89 Then it is over. Of course, this reasoning only works because we are assuming the number has an integer 13th root.

    • @samueldeandrade8535
      @samueldeandrade8535 Місяць тому

      Hahaha, I made some silly calculations. Anyways, using the full cycle of 10 male the calculations far easier.

  • @samueldeandrade8535
    @samueldeandrade8535 Місяць тому

    You made some confusion, didn't you? Because you could divide the number in 3 digits, alternating plus and minus ...

    • @samueldeandrade8535
      @samueldeandrade8535 Місяць тому

      Actually, no ... use the full cycle of 10 (mod 13), making no additional divisions by 13. We have 10¹ = -3 (mod 13) 10² = -4 (mod 13) 10³ = -1 (mod 13) 10⁴ = 3 (mod 13) 10⁵ = 4 (mod 13) 10⁶ = 1 (mod 13) The number is 21 982145 917308 330487 013369 Just sum the digits in each column and multiply by the respective value of 10ⁿ, n=0,...,5, (mod 13), (9+9+3+0)×4 + (8+1+3+1)×3 + (2+7+0+3)×(-1) + (1+3+4+3)×(-4) + (2+4+0+8+6)×(-3) + (1+5+8+7+9)×1 = 21×4 + 13×3 + 12×(-1) + 11×(-4) + 20×(-3) + 30×1 Mod 13: = (-5)×4 + 0×3 + (-1)×(-1) + (-2)×(-4) + (-6)×(-3) + 4×1 = (-5+2)×4 + 0 + 1 + + 18 + 4 = -12 + 1 + 5 + 4 = 1 + 10 = 11 No division of three digit numbers or 6 digit numbers requires. It has to end in 9. So 13k+11 = ...9 This means k ends in 6, so k = 10n+6, 13k+11 = 130n+89 Then it is over. Of course this reasoning only works because we are assuming the number has an integer 13th root.

  • @alexandergroysman2357
    @alexandergroysman2357 Місяць тому

    9 in odd power ends with 9, so our x ends with 9. Now we need a number that in a power 13 reaches 26 digits. So we drop 13 right digits leaving a number with 13 digits. Now find number that in a power 13 is closest to it. (8^13 = 2^13^3): 2^13=8192, 8192^3 ~ 500000000000 12 digits. So our answer would be 89?

  • @xbz24
    @xbz24 Місяць тому

    amazing, i love your vids specially about number theory, keep it up with the good stuff

  • @sarthakjohnsonprasad3909
    @sarthakjohnsonprasad3909 Місяць тому

    Amazing!

  • @beattoedtli1040
    @beattoedtli1040 Місяць тому

    Great. At the end you finished a bit quickly.

  • @kgangadhar5389
    @kgangadhar5389 Місяць тому

    Thanks!

  • @RR-bs9mr
    @RR-bs9mr Місяць тому

    I wonder what is equals

  • @Mystic_hermit
    @Mystic_hermit Місяць тому

    I love how you always give detailed logical proofs of the sub-results used...helps a lot!! Thank you so much for these clear illustrations!

  • @sanyusix8843
    @sanyusix8843 Місяць тому

    I want to be an ass and say, a,b,c,d = 1/4 and it's solved

    • @williamwarren5783
      @williamwarren5783 Місяць тому

      I believe the question is asking for any a,b,c,d the inequality holds

  • @mashalrazavi579
    @mashalrazavi579 Місяць тому

    Very nice equation Thank you Dr Jin